Test #4 PART 1

Réussis tes devoirs et examens dès maintenant avec Quizwiz!

thyroid

-in front of trachea -controls metabolism & assists parathyroid glands

hot area on thryroid scan

-increased fx -shows dark on screen -indicates graves dx, goiter, NEVER CA

thyroidectomy indicated when

-large goiters cause compression onto trachea or esophagus -unresponsive to drug therapy or allergic to iodine used for thyroid scan -possible malignancy (cold spot on thyroid scan or solid mass on US)

tx of thyroid storm

-maintain airway -drug therapy -cardiac monitor & VS -temp control

endoscopic thyroidectomy

-minimally invasive -small nodules <3 cm

Health promotion and prevention of health problems for older adults focus on which three areas?

(1) reduction in diseases and problems (2) increased participation in health promotion activities (3) increased targeted services that reduce health hazards.

hypothalamus hormones

- Corticotropin-releasing hormone (CRH) - Thyrotropin-releasing hormone (TRH) - Growth hormone-releasing hormone (GHRH) - Growth hormone-inhibiting hormone (GHIH)

parathyroid

-4 small glands -regulation of Ca -works on bones, instestines, kidneys

if you can visually see the thyroid

-DO NOT PALPATE it will cause overstimulation, thyroid storm

precautions for thyroid scan

-allergy to iodine -thyroid replacement needs held 4-6 W prior to having -contrainidicated in pregnancy post: flush a couple of time after urinating, usually out of body 48 H

pre op thyroidectomy

-anti-thyroid drugs, iodine, and BB -↑ protein, ↑ carb diet for a couple of weeks before -teach post op care ‣deep breathing ‣support head & neck when moving in bed, ambulating, coughing, prevent stress to suture line ‣normal to be hoarse after for a few D ‣JP drain ‣dressing

hyperthyroidism drugs

-antithryroid drugs ‣methimazole (Tapazole) ‣PTU (propylthiouracil) -iodine preparations ‣saturated solution of potassium iodine SSKI -beta adrenergic blockers ‣supportive therapy

drug therapy for thyroid storm

-antithyroid IV -BB

pituitary

-brain -anterior & posterior lobes ➤anterior lobe ‣releases hormones when stimulated by hypothalamus ➤posterior lobe ‣stores vasopressin, antidiuretic & oxytocin

hypothalamus

-brain -combo of nerves & endocrine tissue -produces regulatory hormones that travel to another endocrine tissue & cause them to secrete a different hormone

hyperthyroidism DC teaching

-check T3, T4, TSH -s/s of hypothyroidism -reduce calorie diet; normal metabolism & will gain weight -regular exercise -eye protection - exopthalmus may not correct (eye drops, elevate HOB, tape eyelids at night, diuretic therapy, steroids)

temp control for thyroid storm

-cooling blankets, ice

negative feedback system adrenal gland

-cortisol level drops -hypothalamus releases coritsol releasing hormone to anterior pituitary, which releases ACTH to adrenal glands to produce cortisol when cortisol come back WNL the hypothalamus stops

cold area on thyroid scan

-decreased fx -hole or blank area on screen -indicates possible CA, need other tests done (Bx)

precautions for Radioactive iodine (RAI)

-decreased protection bc it is a ↑ dose -flush multiple times after urination -don't share towels, toothbrush, sheets -don't be around pregnant women or small kids for at least 1 W -may cause dry mouth, dry throat; ↑ intake of water, offer ice chips

radioactive iodine (RAI) hyperthyroidism

-destroys tissue cells that produce Thyroid hormone -contraindicated in pregnancy -may take up to 2 M -outpatient procedures -multiple precautions -main SE is hypothyroidism (monitor, teach)

methimazole (hyperthyroidism)

-drug of choice -binds & fills receptors to ↓ over stimulation of T3 & T4

hyperthyroidism

-excessive T3 & T4 -affects all systems -temp or perm -normal feedback system fails, causing hypermetabolism & increased SNS

endocrine system is made up of

-glands & hormones

adrenal hormones

-glucocorticoid (cortisol) mineralocorticoid (aldosterone) catecholamine (epi & norepi)

hyperthyroidism manifestations

-heat intolerance -tremors, tachy with palpitations, HTN, murmurs -exhaustion -thin, brittle nails; hair loss, most flush skin -amenorrhea, decreased libido, & fertility -insomnia -increased appetitie with weight loss, diarrhea -N,V -emotional instability, mood swings -dyspnea, increased RR -AFFECTS ALL BODY SYSTEMS

hyperthyroidism nursing interventions (acute care)

-monitor VS Q4H; esp temp (thyroid storm) -maintain quiet, restful, cool environment (HTN, tachy) -balanced rest & activity -diet ↑ in cal, protein, carbs; 4000-5000 cal/day, or will become malnutritioned; avoid spicy and high fiber foods -daily weight, with strict I/O; monitor edema -monitor BM; give antidiarrhetics PRN -allay anxiety-may be hyperactive & irritable -admin eye drops - elevate HOB -good skin care - cool showers, change linens (sweaty) -monitor for arrhythmias & signs of thyroid storm; ventricular dysrythmias (sinus tach & A. Fib)

post op care for thyroid bx

-monitor for bleeding -sxs of puncture esophagus or trachea -may need dressing -may need to limit activity for a short time

thyroid gland general

-most common endocrine d/o -T3 & T4 regulate metabolism, G/D -d/o's include: --hyperthyroidism, hypothyroidism --thyroiditis (inflammation) --thyroid CA

assessing respiratory (post thyroidectomy)

-noisy respirations, such as stridor may indicate swelling, tetany, damage to laryngeal nerve -keep trach kit & suctioning at bedside -humidify O2 -deep breathing

US of thyroid

-ordered if nodules felt on palpation -fluid filled on US are benign -solid are possibly malignant, need further testing (bx) -hyperextend neck -lasts 15-30 minutes -painless -no fasting or sedation -noninvasive

parathyroid hormones

-parathyroid hormone (PTH)

thyroid Bx

-probably follows thyroid scan with cold spot or solid cyst on US -removes small amount of tissue -Open - surgical incision -closed - fine needle aspiration -can be indeterminant results; can not tell from bx and may need to take thyroid

functions of endocrine system

-response to stress & injury -G/D -energy metabolism -reproduction -F/E & acid-base balance

SSKI (saturated solution of potassium iodine) hyperthyroidism

-short term -before surgical procedure -destroys part of thyroid tissue -↓ blood flow to gland & ↓ post op bleeding

BB hyperthyroidism

-supportive therapy -don't affect thyroid at all but will ↓ cardiac sx's

thyroid hormones

-thryroxine (T4) -Triiodothryronine (T3) -Calcitonin

hyperthyroidism surgical therapies

-thyroidectomy -subtotal or total thyroidectomy

3 problems with endocrine system

-too much hormone -too little hormone -receptor site not working properly

adrenal

-top of kidneys -sends hormones that affect all body systems ➤cortex (outer layer) ‣releases cortisol & aldosterone ➤cortisol ‣stress, emotional stability, immune fx, metabolism ➤aldosterone ‣F/E balance ➤Medulla (inner layer) ‣epi & norepi ‣SNS action ‣fight or flight

subtotal or total thyroidectomy

-total will require hormone replacement forever -partial may not need hormone replacement forever -parathyroid is inside thyroid - may be able to imbed into pt to still be able to produce some of hormone

thyroid physical exam

-two handed: stand behind pt -one handed: stand in front of pt -normal: cannot palpate edges -abnormal: enlarged: auscultate for bruit

thyroid storm

-uncontrolled hyperthyroidism -life threatening -common in Graves dx -triggered by: trauma, infection, DKA, pregnancy, overexposure of iodine, post thyroid sx -key manifestations: fever, tachy, systolic HTN

thyroid scan

-useful to determine location, size, & fx of thyroid -used with hypo/hyper, goiter, thyroid CA -radioactive iodine-PO or IV -cold or hot areas

posterior pituitary hormones

-vasopressin (antidiuretic hormone ADH)

evaluate dress for bleeding (post thyroidectomy)

-very vascular -risk of bleeding for 24H -check front of neck (bandage) & back of neck (bandge); bleeding can run to the back of the neck when lying down

PTU (hyperthyroidism)

-will not be used very often -liver toxicity & damage -avoid during pregnancy

risk factors for hyperthyroidism

-women 20-40 Y/O -infection -stressful life event -autoimmune disease

normal TSH

.5-5

The nurse should implement which activity to promote reminiscence among older clients? 1. having storytelling hours 2. setting up pet therapy sessions 3. displaying calendars and clocks 4. encouraging client participation in a pottery class

1 Rationale: clients who like to retell stories or to describe past events need to be provided with the opportunity to do so. this phenomenon is called life review or reminiscence. in a sense, it is a way for older clients to relive and restructure life experiences, and it is a part of achieving ego identity.

Which data indicate to the nurse that a client may be experiencing ineffective coping following the loss of her spouse? 1. constantly neglects personal grooming 2. visits her husbands grave once a month 3. visits the senior citizens once a month 4. frequently looks at snapshots of her husband and family

1 Rationale: coping mechanisms are behaviors that are used to decrease stress and anxiety. in response to a death, ineffective coping is manifested by an extreme behavior that in so instances may be harmful to the individual physically, psychologically, or both. option 1 is indicative of a behavior that identifies an ineffective coping behavior as part of the grieving process

The nurse is planning to feed an older client who is at risk for aspiration of food. During the meal, how should the nurse position the client? 1. upright in a chair 2. on the left side in the bed 3. on the right side in the bed 4. in a low fowler's position, with the legs elevated

1 Rationale: it is preferable to get clients out of bed and sitting in a chair for meals. this position facilitates chewing and swallowing and prevents the reflux of stomach contents and aspiration.

The nurse is preparing to communicate with an older client who is hearing impaired. Which intervention should be implemented initially? 1. stand in front of the client 2. exaggerate the lip movements 3. obtain a sign language interpreter 4. pantomime and write the client notes

1 Rationale: the nurse would ensure that the hearing impaired client can see the nurse when the nurse is speaking by providing adequate lighting and standing in front of the client. the nurse should enunciate words clearly but not exaggerate lip movements. if the client is profoundly hearing impaired and uses signing, a sign language interpreter should be obtained. if a client cannot understand by reading lips, the nurse would try using gestures, pantomiming, or writing notes

A nurse is caring for a patient suffering from Alzheimer's disease. The patient's spouse voices concern about the type and amount of care the patient requires. How can the nurse support this caregiver? Select all that apply. 1 Identify the needs of the caregiver. 2 Teach the caregiver along with the patient. 3 Discourage the caregiver from seeking support from social support systems. 4 Avoid discussing support groups. 5 Provide sources of support for physical and technical requirements.

1, 2, 5 The nurse should support the spouse by identifying the caregiver's needs with respect to the acute care setting, during transition to home, and at home. The nurse should teach both the caregiver and patient, so that they may complement each other. The couple may need support for physical and technical requirements; the nurse should provide information about the related sources. The nurse should encourage the patient to seek help from social support systems with respect to housing, finance, and respite care. The nurse should discuss potential support groups to provide ongoing support and continuing education. Text Reference - p. 52

A student nurse is not comfortable with patient teaching and would rather limit nursing activities to direct patient care only. What appropriate training strategies should the mentor use with this student nurse? Select all that apply. 1 Help the student nurse develop teaching skills. 2 Assign the student nurse to direct patient care only. 3 Remind the student nurse that it takes time to master teaching skills. 4 Verbalize disappointment that the student nurse is not competent. 5 Familiarize the student nurse with various resources that help in patient teaching

1, 3, 5 The student nurse is in the learning stage; therefore the mentor should help the student develop teaching skills. The mentor should explain that with time, teaching skills improve. It would be helpful to familiarize the student nurse with various resources to help in patient teaching. A good nurse should be able to provide both care and teaching, so the mentor should not let the student avoid teaching. The nursing student should not be criticized, but rather helped. Text Reference - p. 51

four components of thyroidism

1. increased hormone production (thyrotoxicosis) 2. enlarged thyroid (goiter) 3. protruding eyes (exopthalmos) 4. skin changes (pretibial myxedema)

normal T3

100-200

When the nurse is collecting data from the older adult, which findings should be considered normal physiological changes? SATA 1. increased heart rate 2. decline in visual acuity 3. decreased resp rate 4. decline in long term memory 5. increased susceptibility to UTI's 6. increased incidence of awakening after sleep onset

2, 5, 6 Rationale: anatomical changes to the eye affect the individual's visual ability, which leads to potential problems with activities of daily living. light adaptation and visual fields are reduced. resp rates are usually unchanged, the heart rate decreases, and the heart valves thicken. age related changes that affect the Urinary tract increase in an older client's susceptibility to UTI's. short term memory may decline with age, but long term memory is usually maintained. changes in sleep patterns are consistent, age related changes. older persons experience an increased incidence of awakening after sleep onset

2. A nurse cares for a client with a hypofunctioning anterior pituitary gland. Which hormones should the nurse expect to be affected by this condition? (Select all that apply.) a. Thyroid-stimulating hormone b. Vasopressin c. Follicle-stimulating hormone d. Calcitonin e. Growth hormone

a. Thyroid-stimulating hormone c. Follicle-stimulating hormone e. Growth hormone

Which client is most likely at risk to become a victim of elder abuse? 1. a 75 year old man with moderate hypertension 2. a 68 year old man with newly diagnosed cataracts 3. a 90 year old woman with advanced parkinson's disease 4. a 70 year old woman with early diagnosed lyme disease

3 Rationale: elder abuse is widespread and occurs among all subgroups of the population. it includes physical and psychological abuse, the misuse of property, and the violation of rights. the typical abuse victim is a woman of advanced age with few social contacts and at least one physical or mental impairment that limits her ability to perform activities of daily living. in addition, the client usually lives alone or with the abuser and depends on the abuser for care

Which intervention should be implemented for the older client with presbycusis who has a hearing loss? 1. speak louder 2. speak more slowly 3. use low pitched tones 4. use high pitched tones

3 Rationale: presbycusis refers to the age related, irreversible, degenerative changes of the inner ear that lead to decreased hearing acuity. as a result of these changes, the older client has a decreased response to high frequency sounds. low pitched tones of voice are more easily heard and interpreted by the older client. speaking softly or slowly is not helpful

A nurse provides education to a patient diagnosed with diabetes mellitus. The patient appears depressed and states, "I don't think I will be able to care for my diabetes." To encourage self-efficacy, what teaching technique should the nurse use? 1 Help the patient deal with difficult goals early in the teaching sessions. 2 Encourage the patient to master complex skills to promote self-efficacy. 3 Proceed from simple to complex content to establish a feeling of success. 4 Inform the patient that repeated failures to acquire skills does not affect self-efficacy.

3 Self-efficacy in patients may be enhanced by motivating them to learn simple skills first and more complex skills later. This helps keep the patient motivated. Setting difficult initial goals may harm the patient's self-efficacy if he fails to meet those goals. Gaining simple skills also increases patients' self-efficacy, so the nurse should not only encourage the patient to gain complex skills. Repeated failure to acquire any skills may decrease self-efficacy. Text Reference - p. 54

barriers for rural older adults

lack of quality care, financial limitations, limited health care workers, lack of transportation, social isolation, and a limited number of health care facilities

normal thyroid uptake of iodine

30-35%

The nurse should plan which to encourage autonomy in the client who is a resident in a long term care facility? 1. choosing his meals 2. decorating his room 3. scheduling his barber appointments 4. allowing the client to choose social activities

4 Rationale: autonomy is the personal freedom to direct ones own life as long as it does not impinge on the rights of others. an autonomous person is capable of rational thought. this individual can identify problems, search for alternatives, and choose solutions that allow for continued personal freedom as long as the rights and property of others are not harmed. the loss of autonomy-and, therefore, independence- is a very real fear among older clients

The nurse is providing an education class to healthy older adults. Which exercise will best promote health maintenance? 1. gardening every day for an hour 2. sculpting once a week for 40 minutes 3. cycling three times a week for 20 minutes 4. walking three to five times a week for 30 mins

4 Rationale: exercise and activity are essential for health promotion and maintenance in the older adult and for achieving an optimal level of functioning. approx half of the physical deterioration of the older client is caused by disuse rather than by the aging process or disease. one of the best exercises for an older adult is walking, with the goal of progressing to 30 min sessions three to five times each week. swimming and dancing are also beneficial

An older client has been prescribed digoxin (Lanoxin). The nurse understands that which age related change would place the client at risk for digoxin toxicity? 1. decreased cough efficiency and vital capacity 2. decreased salivation and GI motility 3. decreased muscle strength and loss of bone density 4. decreased lean body mass and glomerular filtration rate

4 Rationale: the older client is at risk for medication toxicity because of decreased lean body mass and an age associated decreased glomerular filtration rate. although options 1, 2, and 3 identify age related changes that occur in the older client, they are not specifically associated with this risk

What is most appropriate for the nurse to do when interviewing an older patient? A) Ensure all assistive devices are in place. B) Interview the patient and caregiver together. C) Perform the interview before administering analgesics. D) Move on to the next question if the patient does not respond quickly.

A) Ensure all assistive devices are in place. RATIONALE: All assistive devices, such as glasses and hearing aids, should be in place when interviewing an older patient. It is best to interview the patient and caregiver separately to ensure a reliable assessment related to any possible mistreatment. The patient should be free from pain during the assessment and may need extra time to respond to questions.

normal T4

5-12

6. Revise the following medical terms into phrases that a patient with limited health literacy would be able to understand. a. Acute myocardial infarction b. Intravenous pyelogram c. Diabetic retinopathy

A

An appropriate care choice for an older adult who lives with an employed daughter but needs help with activities of daily living is a. adult day care. b. long-term care. c. a retirement center. d. an assisted living facility.

A

Which patient is at higher risk of malnutrition? 1. young adult 2. A frail older adult 3. A patient with aphasia 4. A patient with irritable bowel syndrome

A frail older adult has a higher risk of malnutrition due to inadequate food supply, which may be caused by loneliness, depression, and low income.

A patient with chronic hypertension has a serum potassium level of 10mmol/L. The patient asks the nurse about the relationship between high blood pressure and high serum potassium. If the nurse is unsure about the relationship, how should the nurse respond? 1 Educate the patient about the effects of high blood pressure. 2 Educate the patient about the effects of high serum potassium. 3 Admit not having knowledge about it; reassure the patient that help will be sought to provide the answer. 4 Tell the patient to not worry about it for now; the patient needs to focus on wellness.

A nurse should be aware of knowledge limitations. When the nurse is unsure of a certain question posed by the patient or the caregivers, it should be clarified. If the nurse does not have enough knowledge to answer the question, it should be admitted to the patient. The nurse can then seek help from other members of the health care team or other reliable sources. It would be inappropriate to give information strictly about blood pressure or high potassium. It is the nurse's responsibility to keep the patient informed and to respond appropriately to a patient's questions and concerns. Text Reference - p. 50

A 70-year-old man has just been diagnosed with chronic obstructive pulmonary disease (COPD). At what point should the nurse begin to include the patient's wife in the teaching around the management of the disease? A) As soon as possible B) When the patient requests assistance from his spouse and family C) When the patient becomes unable to manage his symptoms independently D) After the patient has had the opportunity to adjust to his treatment regimen

A) As soon as possible RATIONALE: In the management of chronic illness, it is desirable to include family caregivers in patient education and symptom-management efforts as early in the diagnosis as possible.

Which criterion must a 65-year-old person meet in order to qualify for Medicare funding? A) Being entitled to Social Security benefits Correct B) A documented absence of family caregivers C) A validated need for long-term residential care D) A history of failed responses to standard medical treatments

A) Being entitled to Social Security benefits RATIONALE: In order to qualify for Medicare, an individual must be entitled to receive Social Security benefits. Absence of caregivers and inadequate responses to treatment are not qualification criteria for Medicare, and the program does not cover residential care services.

A nurse who is providing care for an 81-year-old female patient recognizes the need to maximize the patient's mobility during her recovery from surgery. What accurately describes the best rationale for the nurse's actions? A) Continued activity prevents deconditioning. B) Pharmacokinetics are improved by patient mobility. C) Lack of stimulation contributes to the development of cognitive deficits in older adults. D) Regularly scheduled physical rehabilitation provides an important sense of purpose for older patients.

A) Continued activity prevents deconditioning. RATIONALE: Older adults are highly susceptible to deconditioning, a process that can be slowed or prevented by regular physical activity. This consideration supersedes any possible effect on pharmacokinetics, prevention of cognitive deficits, or the patient's sense of purpose.

An 82-year-old woman with chronic heart failure is cared for in the home by her only child. What problem is indicated by the caregiver's failure to provide companionship or social stimulation? A) Neglect B) Sexual abuse C) Abandonment D) Violation of personal rights

A) Neglect RATIONALE: Neglect characteristics: leaving alone for long periods, failing to provide social stimulation and/or companionship; Sexual abuse: nonconsensual sexual contact includes touching inappropriately, forcing sexual contact; Abandonment: desertion by person who's assumed responsibility for providing care or person w/ physical custody; Violation of personal rights: denying right to privacy or make decisions regarding health care or living environment, forcible eviction.

An appropriate care choice for an older adult who lives with an employed daughter but requires help with activities of daily living (ADLs) is: A) adult day care. B) long-term care. C) a retirement center. D) an assisted living facility.

A) adult day care. RATIONALE: Adult day care (ADC) programs provide daily supervision, social activities, and assistance with ADLs for persons who are cognitively impaired and persons who have problems with ADLs. ADC centers provide physical and emotional relief for the caregiver and allow the caregiver to pursue continued employment.

Which assessment findings would alert the nurse to possible elder mistreatment (select all that apply)? A) Agitation B) Depression C) Weight gain D) Weight loss E) Hypernatremia

A, B, D, E [agitation, depression, weight loss, hypernatremia] RATIONALE: Agitation and depression may be manifestations of psychologic abuse or neglect. Hypernatremia may signify dehydration caused by physical neglect. Weight loss, rather than weight gain, is another manifestation of physical neglect.

A 67-year-old woman who has a long-standing diagnosis of incontinence is in the habit of arriving 20 minutes early for church in order to ensure that she gets a seat near the end of a row and close to the exit so that she has ready access to the restroom. Which tasks of the chronically ill is the woman demonstrating (select all that apply)? A) Controlling symptoms B) Preventing social isolation C) Preventing and managing a crisis D) Denying the reality of the problem E) Adjusting to changes in the course of the disease

A, C [Controlling symptoms & Preventing and managing a crisis] RATIONALE: The woman's efforts to ensure that she can continue in her lifestyle of church attendance while accommodating the frequent elimination caused by her health problem are an example of controlling her symptoms and preventing a personal crisis.

A male patient has a history of hypertension and type 1 diabetes mellitus. Because of these chronic illnesses, the patient exercises and eats the healthy diet that his wife prepares for him. Which factors will most likely have a positive impact on his biologic aging (select all that apply)? A) Exercise B) Diabetes C) Social support D) Good nutrition E) Coping resources

A, C, D, E [Exercise, Social support, Good nutrition, Coping resources] RATIONALE: Biologic aging is the progressive loss of function. Obesity, diabetes, hypertension, and cancer are all associated with the effects of aging. Exercise, good nutrition, social support, stress management, and coping resources are all positive factors related to the aging process.

An ethnic older adult may feel a loss of self-worth when the nurse (select all that apply) a. prohibits visits from a faith healer. b. informs the patient about ethnic support services. c. allows a patient to rely on ethnic health beliefs and practices. d. emphasizes that a therapeutic diet does not allow ethnic foods. e. uses a medical interpreter to provide explanations and teaching.

A, D

An 88-year-old male patient with dementia and a fractured hip is admitted to the clinical unit accompanied by his daughter who is his caregiver. The daughter looks tired and disheveled. About 6 months ago she moved in with her father to keep him safe when he started wandering away from his home. She has no money because of her inability to work. She has inadequate information and is concerned about what will happen to her father after this hospitalization. Her brother calls on the phone to tell her what to do but does not come to visit or help out. There is conflict in the family related to decisions about caregiving. She has no respite from caregiving responsibilities and is socially isolated with loss of friends from an inability to have time for herself. What are the best coping strategies to teach this patient's daughter (select all that apply)? a. Keep a journal. b. Get regular exercise. c. Join a support group. d. Go on a weight-loss diet. e. Use humor to relieve stress. f. Take time to read more books.

A,B,C,E

Which statements characterize the teaching-learning process (select all that apply)? a. Learning can occur without teaching. b. Teaching may make learning more efficient. c. Teaching must be well planned to be effective. d. Learning has not occurred when there is no change in behavior. e. Teaching uses a variety of methods to influence knowledge and behavior.

A,B,E

Which teaching strategies should be used when it is difficult to reach the desired goals of the session (select all that apply)? a. DVD b. Role play c. Discussion d. Printed material e. Lecture-discussion f. Web-based programs

A,C,D,F

A patient with a breast biopsy positive for cancer tells the nurse that she has been using information from the Internet to try to make a decision about her treatment choices. In counseling the patient, the nurse knows that (select all that apply) a. the patient should be taught how to identify reliable and accurate information available online. b. all sites used by the patient should be evaluated by the nurse for accuracy and appropriateness of the information. c. most information from the Internet is incomplete and inaccurate and should not be used to make important decisions about treatment. d. the Internet is an excellent source of health information, and online education programs can provide patients with better instruction than is available at clinics. e. the patient should be encouraged to use sites established by universities, the government, or reputable health organizations, such as the American Cancer Society, to access reliable information.

A,E

In what order will the nurse perform these actions when doing a physical assessment for a patient admitted with abdominal pain? (Put a comma and a space between each answer choice [A, B, C, D].) a. Percuss the abdomen to locate any areas of dullness. b. Palpate the abdomen to check for tenderness or masses. c. Inspect the abdomen for distention or other abnormalities. d. Auscultate the abdomen for the presence of bowel sounds.

ANS: C, D, A, B When assessing the abdomen, the initial action is to inspect the abdomen. Auscultation is done next because percussion and palpation can alter bowel sounds and produce misleading findings.

A patient who is actively bleeding is admitted to the emergency department. Which approach is best for the nurse to use to obtain a health history? a. Briefly interview the patient while obtaining vital signs. b. Obtain subjective data about the patient from family members. c. Omit subjective data collection and obtain the physical examination. d. Use the health care provider's medical history to obtain subjective data.

ANS: A In an emergency situation the nurse may need to ask only the most pertinent questions for a specific problem and obtain more information later. A complete health history will include subjective information that is not available in the health care provider's medical history. Family members may be able to provide some subjective data, but only the patient will be able to give subjective information about the bleeding. Because the subjective data about the cause of the patient's bleeding will be essential, obtaining the physical examination alone will not provide sufficient information.

When selecting audiovisual and written materials as teaching strategies, what is important for the nurse to do? a. Provide the patient with these materials before the planned learning experience. b. Ensure that the materials include all the information the patient will need to learn. c. Review the materials before use for accuracy and appropriateness to learning needs and goals. d. Assess the patient's auditory

C

*The nurse who is assessing an older adult with rectal bleeding asks, "Have you ever had a colonoscopy?" The nurse is performing what type of assessment?* a. Focused assessment b. Emergency assessment c. Detailed health assessment d. Comprehensive assessment

ANS: A A focused assessment is an abbreviated assessment used to evaluate the status of previously identified problems and monitor for signs of new problems. It can be done when a specific problem is identified. An emergency assessment is done when the nurse needs to obtain information about life-threatening problems quickly while simultaneously taking action to maintain vital function. A comprehensive assessment includes a detailed health history and physical examination of one body system or many body systems. It is typically done on admission to the hospital or onset of care in a primary care setting.

What factor(s) place older adults at greater risk for elder mistreatment (EM)?

Dementia, decreased social support, inability to perform ADLs, and depression

When assessing for formation of a possible blood clot in the lower leg of a patient, which action should the nurse take first? a. Visually inspect the leg. b. Feel for the temperature of the leg. c. Check the patient's pedal pulses using the fingertips. d. Compress the nail beds to determine capillary refill time.

ANS: A Inspection is the first of the major techniques used in the physical examination. Palpation and auscultation are then used later in the examination.

*The nurse records the following general survey of a patient: "The patient is a 50-year-old Asian female attended by her husband and two daughters. Alert and oriented. Does not make eye contact with the nurse and responds slowly, but appropriately, to questions. No apparent disabilities or distinguishing features." What additional information should the nurse add to this general survey?* a. Nutritional status b. Intake and output c. Reasons for contact with the health care system d. Comments of family members about his condition

ANS: A The general survey also describes the patient's general nutritional status. The other information will be obtained when doing the complete nursing history and examination but is not obtained through the initial scanning of a patient.

After teaching a group of nursing students about drugs used to treat Alzheimer disease, the instructor determines that teaching was successful when the group identifies which of the following as a N-Methyl-D-Aspartic Acid (NMDS)-receptor antagonist? Rivastigmine Galantamine Donepezil Memantine

Memantine

Which of the following is the most consistent and dramatic cognitive impairment seen in dementia? Language Memory Visuospatial Executive functioning

Memory

*The nurse is preparing to perform a focused assessment for a patient complaining of shortness of breath. Which equipment will be needed?* a. Flashlight b. Stethoscope c. Tongue blades d. Percussion hammer

ANS: B A stethoscope is used to auscultate breath sounds. The other equipment may be used for a comprehensive assessment but will not be needed for a focused respiratory assessment.

Which of the following is the primary goal in the treatment of a client diagnosed with delirium? Resolution of confusion Family member verbalization of understanding client's confusion Prevention of recurrence Correction of the underlying physiologic alteration

Resolution of confusion

A client is diagnosed with dementia which has progressed significantly. Which of the following would be the priority for this client? Cognitive interventions Education Safety Support

Safety

*A patient is seen in the emergency department with severe abdominal pain and hypotension. Which type of assessment should the nurse do at this time?* a. Focused assessment b. Subjective assessment c. Emergency assessment d. Comprehensive assessment

ANS: C Because the patient is hemodynamically unstable, an emergency assessment is needed. Comprehensive and focused assessments may be needed after the patient is stabilized. Subjective information is needed, but objective data such as vital signs are essential for the unstable patient.

When admitting a patient who has just arrived on the unit with a severe headache, what should the nurse do first? a. Complete only basic demographic data before addressing the patient's pain. b. Medicate the patient for the headache before doing the health history and examination. c. Take the initial vital signs and then address the headache before completing the health history. d. Inform the patient that the headache will be treated as soon as the health history is completed.

ANS: C The patient priority in this situation will be to decrease the pain level because the patient will be unlikely to cooperate in providing demographic data or the health history until the nurse addresses the pain. However, obtaining information about vital signs is essential before using either pharmacologic or nonpharmacologic therapies for pain control. The vital signs may indicate hemodynamic instability that would need to be addressed immediately.

When assessing a patient's abdomen during the admission assessment, which action should the nurse take first? a. Feel for any masses. b. Palpate the abdomen. c. Listen for bowel sounds. d. Percuss the liver borders.

ANS: C When assessing the abdomen, auscultation is done before palpation or percussion because palpation and percussion can cause changes in bowel sounds and alter the findings. All of the techniques are appropriate, but auscultation should be done first.

During the health history interview, a patient tells the nurse about periodic fainting spells. Which question by the nurse will best elicit any associated clinical manifestations? a. "How frequently do you have the fainting spells?" b. "Where are you when you have the fainting spells?" c. "Do the spells tend to occur at any special time of day?" d. "Do you have any other symptoms along with the spells?"

ANS: D Asking about other associated symptoms will provide the nurse more information about all the clinical manifestations related to the fainting spells. Information about the setting is obtained by asking where the patient was and what the patient was doing when the symptom occurred. The other questions from the nurse are appropriate for obtaining information about chronology and frequency.

Immediate surgery is planned for a patient with acute abdominal pain. Which question by the nurse will elicit the most complete information about the patient's coping-stress tolerance pattern? a. "Can you rate your pain on a 0 to 10 scale?" b. "What do you think caused this abdominal pain?" c. "How do you feel about yourself and your hospitalization?" d. "Are there other major problems that are a concern right now?"

ANS: D The coping-stress tolerance pattern includes information about other major stressors confronting the patient. The health perception-health management pattern includes information about the patient's ideas about risk factors. Feelings about self and the hospitalization are assessed in the self-perception-self-concept pattern. Intensity of pain is part of the cognitive-perceptual pattern.

In clients with Alzheimer disease, neurotransmission is reduced, neurons are lost, and the hippocampal neurons degenerate. Which neurotransmitter is most involved in cognitive functioning? Serotonin Epinephrine Acetylcholine Norepinephrine

Acetylcholine

The nurse plans to complete a physical examination of an alert, older patient. Which adaptations to the examination technique should the nurse include? a. Avoid the use of touch as much as possible. b. Use slightly more pressure for palpation of the liver. c. Speak softly and slowly when talking with the patient. d. Organize the sequence to minimize the position changes.

ANS: D Older patients may have age-related changes in mobility that make it more difficult to change position. There is no need to avoid the use of touch when examining older patients. Less pressure should be used over the liver. Because the patient is alert, there is no indication that there is any age-related difficulty in understanding directions from the nurse.

Is stroke an acute or chronic illness?

Acute

When assessing a client with dementia, the nurse notes that the client is having difficulty identifying common items, such as a ball or book. The nurse interprets this finding as which of the following? Agnosia Apraxia Disturbance of executive function Aphasia

Agnosia

The registered nurse (RN) cares for a patient who was admitted a few hours previously with back pain after falling. Which action can the RN delegate to unlicensed assistive personnel (UAP)? a. Finish documenting the admission assessment. b. Determine the patient's priority nursing diagnoses. c. Obtain the health history from the patient's caregiver. d. Take the patient's temperature, pulse, and blood pressure.

ANS: D The RN may delegate vital signs to the UAP. Obtaining the health history, documentation of the admission assessment, and determining nursing diagnoses require the education and scope of practice of the RN.

*While the nurse is taking the health history, a patient states, "My mother and sister both had double mastectomies and were unable to be very active for weeks." Which functional health pattern is represented by this patient's statement?* a. Activity-exercise b. Cognitive-perceptual c. Coping-stress tolerance d. Health perception-health management

ANS: D The information in the patient statement relates to risk factors and important information about the family history. Identification of risk factors falls into the health perception-health maintenance pattern.

*A nurse performs a health history and physical examination with a patient who has a right leg fracture. Which assessment would be a pertinent negative finding?* a. Patient has several bruised and swollen areas on the right leg. b. Patient states that there have been no other recent health problems. c. Patient refuses to bend the right knee because of the associated pain. d. Patient denies having pain when the area over the fracture is palpated.

ANS: D The nurse expects that a patient with a leg fracture will have pain over the fractured area. The bruising and swelling and pain with bending are positive findings. No other recent health problems is neither a positive nor a negative finding with regard to a leg fracture.

A nurse is providing teaching for a patient with a new diagnosis of hypertension. Which patient characteristic enhances the teaching-learning process? 1 Moderate anxiety 2 High self-efficacy 3 Limited health literacy 4 A humorous outlook

An important characteristic that can enhance the teaching-learning process is the patient's sense of self-efficacy. There is a strong correlation between self-efficacy and outcomes of illness management. Self-efficacy increases when a person gains new skills in managing a threatening situation, but it decreases when the individual experiences repeated failure. The nurse should plan easily attainable goals early in the teaching sessions and proceed from simple to more complex content to establish a feeling of success. Although mild anxiety increases the learner's perceptual and learning abilities, moderate or severe anxiety limits learning. A limited health literacy results in poor patient outcomes, nonadherence with treatment plans, limited self-management skills, and increased health disparities. Humor in the teaching process is important and useful, but the nurse needs to determine when humor is used excessively or inappropriately to avoid reality. Text Reference - p. 49

The nurse is performing an assessment on an older adult client. Which assessment data would indicate a potential complication associated with the skin? 1. Crusting 2. Wrinkling 3. Deepening of expression lines 4. Thinning and loss of elasticity in the skin

Answer: 1 Rationale: The normal physiological changes that occur in the skin of older adults include thinning of the skin, loss of elasticity, deepening of expression lines, and wrinkling. Crusting noted on the skin would indicate a potential complication.

The home health nurse is visiting a client for the first time. While assessing the client's medication history, it is noted that there are 19 prescriptions and several over-the-counter medications that the client has been taking. Which intervention should the nurse take first? 1. Check for medication interactions. 2. Determine whether there are medication duplications. 3. Determine whether a family member supervises medication administration. 4. Call the prescribing primary health care provider (PHCP) and report polypharmacy.

Answer: 2 Rationale: Polypharmacy is a concern in the older client. Duplication of medications needs to be identified before medication interactions can be determined, because the nurse needs to know what the client is taking. Asking about medication administration supervision may be part of the assessment but is not a first action. The phone call to the PHCP is the intervention after all other information has been collected.

The nurse is providing medication instructions to an older client who is taking digoxin daily. The nurse explains to the client that decreased lean body mass and decreased glomerular filtration rate, which are age-related body changes, could place the client at risk for which complication with medication therapy? 1. Decreased absorption of digoxin 2. Increased risk for digoxin toxicity 3. Decreased therapeutic effect of digoxin 4. Increased risk for side effects related to digoxin

Answer: 2 Rationale: The older client is at risk for medication toxicity because of decreased lean body mass and an age-associated decreased glomerular filtration rate. This age-related change is not specifically associated with decreased absorption, decreased therapeutic effect, or increased risk for side effects. Toxicity, or toxic effects, occurs as a result of excessive accumulation of the medication in the body.

The home care nurse is visiting an older client whose spouse died 6 months ago. Which behaviors by the client indicates effective coping? Select all that apply 1. Neglecting personal grooming 2. Looking at old snapshots of family 3. Participating in a senior citizens program 4. Visiting the spouse's grave once a month 5. Decorating a wall with the spouse's pictures and awards received

Answer: 2, 3, 4, 5 Rationale: Coping mechanisms are behaviors used to decrease stress and anxiety. In response to a death, ineffective coping is manifested by an extreme behavior that in some cases may be harmful to the individual physically or psychologically. Neglecting personal grooming is indicative of a behavior that identifies ineffective coping in the grieving process. The remaining options identify appropriate and effective coping mechanisms.

The long-term care nurse is performing assessments on several of the residents. Which are normal age-related physiological changes the nurse should expect to note? Select all that apply. 1. Increased heart rate 2. Decline in visual acuity 3. Decreased respiratory rate 4. Decline in long-term memory 5. Increased susceptibility to urinary tract infections 6. Increased incidence of awakening after sleep onset

Answer: 2, 5, 6 Rationale: Anatomical changes to the eye affect the individual's visual ability, leading to potential problems with activities of daily living. Light adaptation and visual fields are reduced. Although lung function may decrease, the respiratory rate usually remains unchanged. Heart rate decreases and heart valves thicken. Age-related changes that affect the urinary tract increase an older client's susceptibility to urinary tract infections. Shortterm memory may decline with age, but long-term memory usually is maintained. Change in sleep patterns is a consistent, age-related change. Older persons experience an increased incidence of awakening after sleep onset.

The nurse is providing an educational session to new employees, and the topic is abuse of the older client. The nurse helps the employees identify which client as most typically a victim of abuse? 1. A man who has moderate hypertension 2. A man who has newly diagnosed cataracts 3. A woman who has advanced Parkinson's disease 4. A woman who has early diagnosed Lyme disease

Answer: 3 Rationale: Elder abuse includes physical, sexual, or psychological abuse; misuse of property; and violation of rights. The typical abuse victim is a woman of advanced age with few social contacts and at least 1 physical or mental impairment that limits her ability to perform activities of daily living. In addition, the client usually lives alone or with the abuser and depends on the abuser for care.

The nurse is performing an assessment on an older client who is having difficulty sleeping at night. Which statement by the client indicates the need for further teaching regarding measures to improve sleep? 1. "I swim 3 times a week." 2. "I have stopped smoking cigars." 3. "I drink hot chocolate before bedtime." 4. "I read for 40 minutes before bedtime."

Answer: 3 Rationale: Many nonpharmacological sleep aids can be used to influence sleep. However, the client should avoid caffeinated beverages and stimulants such as tea, cola, and chocolate. The client should exercise regularly, because exercise promotes sleep by burning off tension that accumulates during the day. A 20- to 30-minute walk, swim, or bicycle ride 3 times a week is helpful. Smoking and alcohol should be avoided. Reading is also a helpful measure and is relaxing.

The nurse is providing instructions to the assistive personnel (AP) regarding care of an older client with hearing loss. What should the nurse tell the AP about older clients with hearing loss? 1. They are often distracted. 2. They have middle ear changes. 3. They respond to low-pitched tones. 4. They develop moist cerumen production

Answer: 3 Rationale: Presbycusis refers to the age-related irreversible degenerative changes of the inner ear that lead to decreased hearing ability. As a result of these changes, the older client has a decreased response to high-frequency sounds. Low-pitched voice tones are heard more easily and can be interpreted by the older client. Options 1, 2, and 4 are not accurate characteristics related to aging.

The visiting nurse observes that the older male client is confined by his daughterin-law to his room. When the nurse suggests that he walk to the den and join the family, he says, "I'm in everyone's way; my daughter-in-law needs me to stay here." Which is the most important action for the nurse to take? 1. Say to the daughter-in-law, "Confining your father-in-law to his room is inhumane." 2. Suggest to the client and daughter-in-law that they consider a nursing home for the client. 3. Say nothing, because it is best for the nurse to remain neutral and wait to be asked for help. 4. Suggest appropriate resources to the client and daughter-in-law, such as respite care and a senior citizens center

Answer: 4 Rationale: Assisting clients and families to become aware of available community support systems is a role and responsibility of the nurse. Observing that the client has begun to be confined to his room makes it necessary for the nurse to intervene legally and ethically, so option 3 is not appropriate and is passive in terms of advocacy. Option 2 suggests committing the client to a nursing home and is a premature action on the nurse's part. Although the data provided tell the nurse that this client requires nursing care, the nurse does not know the extent of the nursing care required. Option 1 is incorrect and judgmental

The nurse is caring for an older client in a long-term care facility. Which action contributes to encouraging autonomy in the client? 1. Planning meals 2. Decorating the room 3. Scheduling haircut appointments 4. Allowing the client to choose social activities

Answer: 4 Rationale: Autonomy is the personal freedom to direct one's own life as long as it does not impinge on the rights of others. An autonomous person is capable of rational thought. This individual can identify problems, search for alternatives, and select solutions that allow continued personal freedom as long as others and their rights and property are not harmed. Loss of autonomy, and therefore independence, is a real fear of older clients. The correct option is the only one that allows the client to be a decision maker.

Aphasia

Aphasia is a disorder resulting from brain injury or damage that results in impairments in communication, such as the inability to read, write, or speak effectively.

Which of the following terms describes an impaired ability to execute a voluntary movement despite normal muscle function? Dyslexia Ataxia Apraxia Dysphagia

Apraxia

Genetics appears to play a role in the development of early-onset Alzheimer disease (AD). What type of genetic transmission is seen in AD? X-linked recessive X-linked dominant Autosomal dominant Autosomal recessive

Autosomal dominant

Examples of primary prevention strategies include a. colonoscopy at age 50. b. avoidance of tobacco products. c. teaching the importance of exercise to a patient with hypertension. d. intake of a diet low in saturated fat in a patient with high cholesterol.

B

What is the best example of documentation of patient teaching about wound care? a. "The patient was instructed about care of wound and dressing changes." b. "The patient demonstrated correct technique of wound care following instruction." c. "The patient and caregiver verbalize that they understand the purposes of wound care." d. "Written instructions regarding wound care and dressing changes were given to the patient."

B

Which action by a nurse demonstrates an empathetic approach to patient teaching? a. Assesses the patient's needs before developing the teaching plan b. Provides positive nonverbal messages that promote communication c. Reads and reviews educational materials before distributing them to patients and families d. Overcomes personal frustration when patients are discharged before teaching is complete

B

Which strategy should the nurse use to promote a patient's self-efficacy during the teaching-learning process? a. Emphasize the relevancy of the teaching to the patient's life. b. Begin with concepts and tasks that are easily learned to promote success. c. Provide stimulating learning activities that encourage motivation to learn. d. Encourage the patient to learn independently without instruction from others.

B

A nurse is caring for an adult who sustained a severe traumatic brain injury following a motor vehicle accident. Once the patient recovers from the acute aspects of this injury and is no longer ventilator-dependent, discharge planning would include that this patient will be transferred to what type of practice setting? A) Assisted living B) Acute rehabilitation C) Long-term acute care D) Skilled nursing facility

B) Acute rehabilitation RATIONALE: Acute rehabilitation practice settings provide a post-acute level of care specializing in therapies for patients with neurologic or physical injuries, such as those with head trauma, spinal cord injury, or stroke.

What should be included when planning care for an older adult? A) Patient priorities should be the only focus of care. B) Additional time related to declining energy reserves. C) Reduction of disease and problems should be the focus. D) Tobacco cessation will help the patient cope with other illnesses.

B) Additional time related to declining energy reserves. RATIONALE: Additional time required w/ older patients w/ declining energy reserves. Pt. priorities considered to best meet pt. needs but will not be only focus of care. Focusing on strengths and abilities as well as physical and mental status will facilitate goal setting to reduce disease or problems. As with all patients, safety is a primary concern, and decreasing tobacco use will improve all of the patient's body functioning.

A 60-year-old female patient has had increased evidence of dementia and physical deterioration. What would be the best assistance to recommend to her caregiver husband who is exhausted? A) Long-term care B) Adult day care Correct C) Home health care D) Homemaker services

B) Adult day care RATIONALE: Adult day care provides social, recreational, and health-related services in safe, community-based environment that would keep pt. safe & decrease stress on husband. Long-term care used when pt. has rapid deterioration, caregiver's unable to continue care, and there's alteration in or loss of family support system. Home health care when there's supportive caregiver involvement for pt. w/ health needs. Homemaker services provide services, not pt. care.

Example of primary prevention strategies include: A) colonoscopy at age 50. B) avoidance of tobacco products. C) intake of a diet low in saturated fat in a patient with high cholesterol. D) teaching the importance of exercise to a patient with hypertension.

B) avoidance of tobacco products. RATIONALE: Primary prevention refers to measures such as proper diet, suitable exercise, and timely immunizations that prevent the occurrence of a specific disease. Colonoscopy is a diagnostic test and therefore secondary prevention. In C and D, patients already have condition (high cholesterol and HTN) so it cannot be primary prevention.

What is biologic aging?

Biologic aging is the progressive loss of function.

Signs of abuse: sexual

Bruises on the breast, anal bleeding, STD, and genital infection

Signs of abuse: physical

Bruises, bilateral injuries, and oversedation

A 68-year-old female patient admitted with a stroke 3 days ago has weakness on her right side. She states, "I will never be able to take care of myself. I don't want to go to therapy this afternoon." After listening to her, which statement would be included as part of a motivational interview? a. "Why not?" b. "If you go to therapy, I'll give you a back rub when you get back." c. "I know you are tired but look how much easier walking was today than it was last week." d. "Well, with that attitude, you will have trouble. The doctor ordered therapy because he thought it would help."

C

A patient with diabetes tells the nurse that he cannot see any reason to change his eating habits because he is not overweight. What action does the nurse determine as the most appropriate for a patient at this stage of the Transtheoretical Model of Health Behavior Change? a. Help the patient set priorities for managing his diabetes. b. Arrange for the dietitian to describe what dietary changes are needed. c. Explain that dietary changes can help prevent long-term complications of diabetes. d. Emphasize that he must change behaviors if he is going to control his blood glucose levels.

C

An important nursing action to help a chronically ill older adult is to a. avoid discussing future lifestyle changes. b. ensure the patient that the condition is stable. c. treat the patient as a competent manager of the disease. d. encourage the patient to "fight" the disease as long as possible.

C

Nursing interventions directed at health promotion in the older adult are primarily focused on a. disease management. b. controlling symptoms of illness. c. teaching positive health behaviors. d. teaching about nutrition to enhance longevity.

C

The nurse assesses a 48-year-old male patient and his family for learning needs related to the myocardial infarction the patient experienced 2 days ago. While doing an assessment, the nurse finds out that the patient's father died at age 52 years from a myocardial infarction. Which assessment area will influence the teaching plan for this patient and family? a. Learner characteristics b. Physical characteristics c. Psychologic characteristics d. Sociocultural characteristics

C

Ageism is characterized by: A) Denial of negative stereotypes regarding aging. B) Positive attitudes toward the elderly based on age. C) Negative attitudes toward the elderly based on age. D) Negative attitudes toward the elderly based on physical disability.

C) Negative attitudes towards the elderly based on age. RATIONALE: Ageism is a negative attitude based on age.

Aging primarily affects the _________of drugs. A) excretion B) absorption C) metabolism D) distribution

C) metabolism RATIONALE: Because the liver mass shrinks and hepatic blood flow and enzyme activity decrease in older adults, metabolism of drugs drops 1/2 to 2/3 of the rate of young adults. This increases the chance of drug toxicity and adverse drug events.

Nursing interventions directed at health promotion in the older adult are primarily focused on: A) disease management. B) controlling symptoms of illness. C) teaching positive health behaviors. D) teaching regarding nutrition to enhance longevity.

C) teaching positive health behaviors. RATIONALE: A high value should be placed on health promotion and positive health behaviors.

An important nursing action to help a chronically ill older adult is to: A) avoid discussing future lifestyle changes. B) assure the patient that the condition is stable. C) treat the patient as a competent manager of the disease. D) encourage the patient to "fight" the disease as long as possible.

C) treat the patient as a competent manage of the disease. RATIONALE: Chronically ill older adults should understand and manage their own health. Self-management is the individual's ability to manage his or her symptoms, treatment, physical and psychosocial consequences, and lifestyle changes in response to living with a long-term disorder.

A characteristic of a chronic illness is that it (select all that apply) a. has reversible pathologic changes. b. has a consistent, predictable clinical course. c. results in permanent deviation from normal. d. is associated with many stable and unstable phases. e. always starts with an acute illness and then progresses slowly.

C, D

A characteristic of a chronic illness is that it (select all that apply): A) has reversible pathologic changes. B) has a consistent, predictable clinical course. C) results in permanent deviation from normal. D) is associated with many stable and unstable phases. E) always starts with an acute illness and then progresses slowly.

C, D [has a consistent, predictable clinical course; results in permanent deviation from normal] RATIONALE: Characteristics of chronic illness include: permanent impairments or deviations from normal, irreversible pathologic changes, residual disability, requirements for special rehabilitation, and need for long-term medical and/or nursing management. Chronic illness may have stable and unstable periods.

hypocalcemia s/s

CATS Convulsions Arrhythmia Tetany Spasms and strider numbness around mouth, lips, toes, spasms

Among older Americans in the United States a. more than 30% live in nursing homes. b. women are less likely to live in poverty. c. the number of those who completed college is lower than in previous decades. d. those 85 years or older account for the fastest growing segment of the population.

D

Older adults who become ill are more likely than younger adults to a. report symptoms to their health care providers. b. refuse to carry out lifestyle changes to promote recovery. c. seek medical attention because of limitations on their lifestyle. d. alter their daily living activities to accommodate new symptoms.

D

A 74-year-old patient admitted with an irregular cardiac rhythm has a much lower blood level of his home medication than expected. Which common cause of medication errors made by older adults should be suspected? A) Shorter drug half-life leading to medication overdose B) Decreased use of nonprescription over-the-counter drugs C) Improved tolerance of adverse effects of prescribed drugs D) Lack of financial resources to obtain prescribed medications

D) Lack of financial resources to obtain prescribed medications RATIONALE: Common causes of medication errors by older adults include poor eyesight, forgetting to take drugs, use of nonprescription over-the-counter drugs, use of medications prescribed for someone else, lack of financial resources to obtain prescribed medication, failure to understand instructions or importance of drug treatment, and refusal to take medication because of undesirable side effects. Drug half-life is increased in older adults.

Older adults who become ill are more likely than younger adults to: A) complain about the symptoms of their problems. B) refuse to carry out lifestyle changes to promote recovery. C) seek medical attention because of limitations on their lifestyles. D) alter their daily living activities to accommodate new symptoms.

D) alter their daily living activities to accommodate new symptoms. RATIONALE: Older adults may underreport symptoms and treat these symptoms by altering their functional status.

An ethnic older adult may feel a loss of self-worth when the nurse: A) informs the patient about ethnic support services. B) allows a patient to rely on ethnic health beliefs and practices. C) has to use an interpreter to provide explanations and teaching. D) emphasizes that a therapeutic diet does not allow ethnic foods.

D) emphasizes that a therapeutic diet does not allow ethnic foods. RATIONALE: An older adult with strong ethnic and cultural beliefs may experience loss of self if nurses deny or ignore ethnic and cultural practices and behaviors.

Which teaching strategies should the nurse plan to use for a patient from the Baby Boomer generation (select all that apply)? a. Podcast b. Role playing c. Group teaching d. Lecture-discussion e. A game or game system f. Patient education TV channels

D,F

Deconditioning

Deconditioning occurs as a result of unstable acute medical conditions, environmental barriers that limit mobility, and a lack of motivation to stay in condition

Depression in older adults

Depression in older adults arises due to loss of self-esteem and changing life situations, such as the ability to care for oneself. It manifests as loss of sleep, weight loss, decreased memory, nervousness, lack of energy, and inability to concentrate. A patient with depression has higher risk of frailty.

Signs of abuse: psychologic

Depression, withdrawn behavior, and agitation

Changes that are found during the mental status examination of a client diagnosed with delirium include which of the following? Increased attention to detail Difficulty focusing Clear memory No impairment of consciousness

Difficulty focusing

A client diagnosed with Alzheimer disease (AD) has decided that he is more comfortable naked than in clothes. This would be documented as which of the following? Cognitive reserve Hypersexuality Disinhibition Bradykinesia

Disinhibition

Downward trajectory meaning

Downward trajectory of a chronic illness happens when the patient has severe symptoms with increased disability and progressive deterioration.

negative feedback system

EX: drop in thyroid hormones (T3 & T4), stimulates hypothalamus to release thyroid releasing hormone to anterior pituitary gland, which releases TSH, which causes thyroid to release T3 & T4. when T3 & T4 come back WNL, the hypothalamus will shut off

Xerostomia is common in older adults. This condition impairs which area of activities of daily living (ADLs)? Eating Bathing Sleeping Walking

Eating

An older adult client with dementia is experiencing sleep problems and becomes restless when he is fatigued. Which of the following would be helpful? Maintaining a high-stimulation environment Ensuring the client has an afternoon nap Having the client rest in a reclining chair during the afternoon Administering a sedative-hypnotic long-term

Ensuring the client has an afternoon nap

Which factors have a positive impact on biologic aging?

Exercise, good nutrition, social support, stress management, and coping resources are all positive factors related to the aging process.

Who is responsible for the majority of reported elder mistreatment cases?

Family members

When describing the dementia associated with Huntington disease, the nurse understands that the problems involving behavior and attention arise from a disruption in which lobe of the brain? Frontal Temporal Occipital Parietal

Frontal

The nurse is caring for a client receiving an acetylcholinesterase inhibitor (AChEI) for treatment of dementia. What is the primary side effect of AChEIs? Muscle cramps Headache Dizziness Gastrointestinal distress

Gastrointestinal distress

The nurse is developing a teaching plan for the patient from a different culture. The priority of the nurse is to assess the patient's: 1 National origin 2 Family members 3 Religious practices 4 Health and illness beliefs

Health and illness beliefs assist the nurse to tailor the patient teaching to cultural needs. This avoids stereotyping patients. National origin is not pertinent to tailoring the patient teaching to cultural needs. Family members may play an important role for the patient; however, the teaching plan is developed based on patient cultural needs. Religious practices may play a role during illness; however, the focus is on health and illness beliefs. Health practices, beliefs, and behaviors are influenced by one's cultural traditions. The Joint Commission requires patient teaching is tailored to the patient's cultural needs. Text Reference - p. 54 TEST-TAKING TIP: Attempt to select the answer that is most complete and includes the other answers within it. For example, a stem might read, "A child's intelligence is influenced by:" and three options might be genetic inheritance, environmental factors, and past experiences. The fourth option might be multiple factors, which is a more inclusive choice and therefore the correct answer.

Which of the following is the priority when caring for a client with delirium? Identifying the cause Maintaining fluid balance Promoting adequate rest Protecting from injury

Identifying the cause

Which technique is most appropriate when using motivational interviewing with a patient who tells the nurse that he or she is ready to stop smoking cigarettes? 1 Confirm that the patient is serious about stopping smoking. 2 Encourage the patient to consider an organized group smoking cessation program. 3 Focus on the patient's strengths to support his or her optimism that he or she can successfully stop smoking. 4 Ask a prescribed set of questions to increase the patient's awareness of his or her cigarette smoking behaviors.

Motivational interviewing uses nonconfrontational interpersonal communication techniques to motivate patients to change behavior. Motivational support is tailored to the stage of change the patient is experiencing. This patient is in the preparation stage of change and needs the nurse's support of the belief that he or she can stop smoking successfully. Confirming that the patient is serious about smoking cessation, encouraging the patient to consider a support group, and asking a set of questions to increase the patient's awareness regarding smoking behaviors do not reflect motivational interviewing techniques. Text Reference - p. 49

Which ailments are associated with biologic aging?

Obesity, diabetes, hypertension, and cancer are all associated with the effects of aging.

A nurse is preparing a presentation for families of clients with Alzheimer disease. When describing the underlying mechanisms associated with this condition, which of the following would the nurse most likely include? (Select all that apply.) Oxidative stress Beta-amyloid destruction Neurofibrillary tangles Enhanced neurotransmission Inflammation

Oxidative stress Neurofibrillary tangles Inflammation

results for thryroid scan usually take

PO - 24 H IV - 2-4 H

Signs of: attempting to normalize interactions with others

PT with a chronic lung problem who has stopped walking suddenly to catch breath appears to be inspecting a plant to hide his or her chronic illness.

Signs of: preventing and managing a crisis

PT with an acute exacerbation of disease symptoms by adhering to a treatment regimen

Signs of: attempting to normalize interactions with others

PT with aphasia who is still willing to go out in public

Signs of: controlling symptoms

PT with irritable bowel syndrome is attending events at which restrooms are available near the seating area

An older adult suffering from asthma is particularly good at memorizing information. Which strategy would be effective for the nurse to use to teach this patient? 1 Role play 2 Pictures and books 3 Internet access 4 Support groups

Pictures and books would be most effective for this patient's education because the patient believes in memorizing information, and books and pictures facilitate memorization. Role playing and support groups are helpful for patients who interact with groups, and who believe in self-directed learning. Providing internet access is helpful for technically comfortable patients. Text Reference - p. 56

Home health care

Provided to homebound patients. The caregiver cares for the patients who are in need of acute health care.

A nurse suspects a chronic illness patient is the victim of elder mistreatment. What is the priority action of the nurse?

Report the findings to adult protective services.

A nurse is planning to teach a group of patients about breast cancer self-screening techniques. What teaching strategy is best for the nurse to use in this situation? 1 Role play 2 Discussion 3 Demonstration 4 Lecture-discussion

Self-screening for breast cancer is a skill; therefore, it is best taught through demonstration. A role play is suitable for examining attitudes and behavior. A discussion involves an exchange of points of view between two individuals. A lecture-discussion is an exchange of points of view among a large group of people. Text Reference - p. 57

Elderly Assessment of ADLs: SPICES

Sleep disorders Problems with eating Incontinence Confusion Evidence of Falls Skin Breakdown

hyperthyroidism labs

T3 >200 T4 >12 TSH - depends on cause usually >5, unless graves dx where it won't be elevated

A cardiac patient spends much time traveling due to work obligations. The patient wishes to obtain healthcare services, including consultation with the patient's primary health care provider, even when in a remote area. What health care technology should the nurse recommend to this patient? 1 Telehealth 2 Websites 3 Wi-Fi devices 4 Wireless technology

Telehealth is a health care technology through which one can obtain health-related services and information via telecommunication technology. The patient and the healthcare provider can connect with each other and share information even when located in different geographical locations. Websites and Wi-Fi devices require internet access. Wireless technology helps monitor patient parameters, but would not necessarily be useful in this case. Text Reference - p. 58

Which teaching strategy would be best to offer to a 20-year-old patient? 1 Lecture or books 2 Websites or podcasts 3 Television or pamphlets 4 Role play or support group

The 20-year-old patient is likely to be a multitasker who prefers interactive and virtual environments and has a short attention span. Internet, websites, podcasts, or video game systems are recommended to teach Millenials health behaviors. Veterans (born before 1946) prefer lecture and book strategies. Baby Boomers (born 1946-1964) prefer lecture and discussion, educational TV, or printed materials. Generation X (born 1965-1980) prefer group teaching, support groups, role playing, and Internet-based education materials. Text Reference - p. 55

The nurse is interviewing the caregiver of a patient undergoing hemodialysis three times per week. What could be stressors for the caregiver? 1 Having a friend accompany the patient to the dialysis unit 2 Inability to meet personal self-care needs 3 Ability to recognize the progressive nature of the illness 4 Visits by members of community organizations

The inability to meet personal self-care needs is a major stressor for the caregiver. Having a friend accompany the patient to the dialysis unit and visits by members of community organizations will provide respite for the caregiver. The ability to recognize the progressive nature of the illness will be helpful in bonding with the patient and in providing adequate care. Text Reference - p. 51

When a patient asks a question about the causes of a diagnosis and the nurse is unsure of the correct answer, the nurse realizes that it is important to avoid doing what? 1 Seeking help from coworkers 2 Providing vague explanations 3 Admitting that the nurse is unsure 4 Seeking help from reliable sources

The nurse should avoid giving vague explanations if he or she is unaware of the causes of the development of the patient's condition. Giving vague explanations might confuse the patient. In addition, the patient may not trust the nurse later. The nurse should seek help from co-workers and reliable sources to learn about the patient's situation, and then inform the patient. Admitting that she is unsure is not a problem; it would show that she is being genuine. Text Reference - p. 51

Preventing and managing a crisis consists of what?

The patient should be educated on early detection of symptoms preceding a crisis and how to manage the situation if a crisis occurs

Age-associated memory impairment

The patient shows symptoms like memory lapse or benign forgetfulness.

Based on adult learning principles, which situation indicates that the patient is ready to learn about taking enoxaparin (Lovenox) injections at home? 1 The patient is requesting pain medication. 2 The patient is too tired to stay awake to watch the teaching DVD. 3 The patient wants to practice before actually injecting him- or herself with the needle. 4 The patient is nervous and says he or she cannot do it when he or she picks up the bottle of medication.

The patient wanting to practice before injecting him- or herself is demonstrating the learner's orientation to learning by seeking out a resource for this stage of learning. The patient requesting pain medication and the tired patient demonstrate they do not have the readiness to learn. The nervous, unconfident patient demonstrates that the learner's self-concept is in need of encouragement and more teaching is needed. Other adult learning principles include the learner's need to know, prior experiences, and motivation to learn. TEST-TAKING TIP: Relax during the last hour before an exam. Your brain needs some recovery time to function effectively. Text Reference - p. 54

other causes of hyperthyroidism

Toxic multinodular goiter - generalized enlargement of thyroid (most often benign) large gland but won't have other sx's Exogenous hyperthyroidism - excessive use of thyroid replacement hormones

Which type of hallucination is most commonly seen in clients diagnosed with dementia? Gustatory Auditory Visual Olfactory

Visual

10. A nurse cares for a client who is prescribed a 24-hour urine collection. The unlicensed assistive personnel (UAP) reports that, while pouring urine into the collection container, some urine splashed his hand. Which action should the nurse take next? a. Ask the UAP if he washed his hands afterward. b. Have the UAP fill out an incident report. c. Ask the laboratory if the container has preservative in it. d. Send the UAP to Employee Health right away.

a. Ask the UAP if he washed his hands afterward.

A nurse is caring for a patient who suffers from alcoholism. The patient has limited literacy. Which teaching interventions would be appropriate for this patient? 1 Encouraging the patient to enroll in adult literacy programs 2 Tailoring the teaching aids to the patient's literacy abilities 3 Discussing studies that describe the impact of literacy on patient outcomes 4 Ensuring that the patient teaching materials are at a sixth grade reading level or higher

When educating an illiterate patient, the nurse should tailor the teaching aids to the patient's literacy abilities. Doing so may help the patient to understand the teaching better. Enrolling in an adult literacy program would help the patient increase the literacy level, but it would not help solve the immediate problem. It is not an appropriate time to discuss research studies; the patient needs a teaching technique that will meet the client's current learning needs. Low literacy results in poor patient outcomes, non-adherence to treatment plans, and higher health disparities. The patient teaching materials should be written at a fifth grade or lower reading level. Text Reference - p. 54

A frail, elderly widow is admitted to the hospital after sustaining a fall. The client lives alone and has no living relatives. After cognitive testing reveals mild cognitive impairment, the interdisciplinary team on the Acute Care for the Elderly Unit recommends long-term care placement and that a durable power of attorney for health care (DPOA-HC) be established. When the client seems confused over what a DPOA-HC's responsibilities are, the nurse responds that: a. "A DPOA-HC is a person you name to make health care decisions for you when you can't make them for yourself." b. "A DPOA-HC is a person you trust to make financial decisions for you and to manage your money." c. "A DPOA-HC is a person appointed by the court to make sure you get good care and to manage your affairs." d. "A DPOA-HC is a person who is appointed by the court to make nursing home placement decisions for your care."

a. "A DPOA-HC is a person you name to make health care decisions for you when you can't make them for yourself."

When a cognitively impaired, wealthy, white client is noted to have burns on her upper back, her son states that the patient burned herself when attempting to shower. Which statement by a member of the team reflects a need for further education on elder abuse? (Select all that apply.) a. "She is wealthy; abuse does not happen in people of financial means." b. "Even if we are not sure, we are legally bound to report our suspicions." c. "We need to consider that most abusers are either adult children or spouses." d. "Her cognitive deficiencies put her at risk for elder abuse." e. "The client is white and race places an important role in who is likely to be abused."

a. "She is wealthy; abuse does not happen in people of financial means." e. "The client is white and race places an important role in who is likely to be abused."

A nurse working in an emergency department is caring for an 89-year-old woman who was brought to the hospital by her daughter for a fracture of the right arm. The woman is wheelchair dependent and lives with her widowed daughter who is the primary caregiver. The daughter states that her mother got up out of the wheelchair unassisted to go to the bathroom and fell. The patient cannot recall the circumstances of the fall. The patient is weeping and cradling her right arm. The patient's history reveals two previous wrist fractures over the course of the past year. The nurse notes several large ecchymotic areas on the right hand and left arm and on the left side of the body and the back. The ecchymoses are in various stages of healing. Upon assessment, the patient is non-weight-bearing (NWB). The nurse suspects physical abuse based on which of the following findings? (Select all that apply.) a. Bruises are in various stages of healing. b. The fracture is inconsistent with the patient's functional ability. c. Caregiver suffering stress from caring for a functionally-dependent individual. d. Patient is crying. e. Patient has a history of previous wrist fractures.

a. Bruises are in various stages of healing. b. The fracture is inconsistent with the patient's functional ability.

An older woman tearfully tells a nurse, "I must buy my neighbor all of his groceries, or he will not drive me to the store or the doctor." This is an example of which type of elder mistreatment? a. Financial exploitation b. Psychological abuse c. Caregiver neglect d. Abandonment

a. Financial exploitation

A nurse is preparing education for a group of older adults and caregivers at a senior center on elder abuse. The nurse is preparing to discuss seniors who are more likely to be abused or neglected. The nurse includes which of the following? (Select all that apply.) a. Individuals with cognitive impairment b. Individuals who abused the caregiver earlier in life c. Individuals who live in an institutional setting d. Individuals who are married and living with a spouse e. Men living alone or in a household with family members

a. Individuals with cognitive impairment b. Individuals who abused the caregiver earlier in life c. Individuals who live in an institutional setting

2. A nurse assesses clients with potential endocrine disorders. Which clients are at high risk for hypopituitarism? (Select all that apply.) a. A 20-year-old female with benign pituitary tumors b. A 32-year-old male with diplopia c. A 41-year-old female with anorexia nervosa d. A 55-year-old male with hypertension e. A 60-year-old female who is experiencing shock f. a 68-year-old male who has gained weight recently

a. A 20-year-old female with benign pituitary tumors c. A 41-year-old female with anorexia nervosa d. A 55-year-old male with hypertension e. A 60-year-old female who is experiencing shock

. A nurse assesses clients with potential endocrine disorders. Which clients are at high risk for adrenal insufficiency? (Select all that apply.) a. A 22-year-old female with metastatic cancer b. A 43-year-old male with tuberculosis c. A 51-year-old female with asthma d. A 65-year-old male with gram-negative sepsis e. A 70-year-old female with hypertension

a. A 22-year-old female with metastatic cancer b. A 43-year-old male with tuberculosis d. A 65-year-old male with gram-negative sepsis

3. A nurse assesses clients who have endocrine disorders. Which assessment findings are paired correctly with the endocrine disorder? (Select all that apply.) a. Excessive thyroid-stimulating hormone Increased bone formation b. Excessive melanocyte-stimulating hormone Darkening of the skin c. Excessive parathyroid hormone Synthesis and release of corticosteroids d. Excessive antidiuretic hormone Increased urinary output e. Excessive adrenocorticotropic hormone Increased bone resorption

a. Excessive thyroid-stimulating hormone Increased bone formation b. Excessive melanocyte-stimulating hormone Darkening of the skin

4. A nurse assesses a client who is prescribed a medication that stimulates beta1 receptors. Which assessment finding should alert the nurse to urgently contact the health care provider? a. Heart rate of 50 beats/min b. Respiratory rate of 18 breaths/min c. Oxygenation saturation of 92% d. Blood pressure of 144/69 mm Hg

a. Heart rate of 50 beats/min Stimulation of beta1 receptor sites in the heart has positive chronotropic and inotropic actions. The nurse expects an increase in heart rate and increased cardiac output. The client with a heart rate of 50 beats/min would be cause for concern because this would indicate that the client was not responding to the medication.

1. A nurse cares for clients with hormone disorders. Which are common key features of hormones? (Select all that apply.) a. Hormones may travel long distances to get to their target tissues. b. Continued hormone activity requires continued production and secretion. c. Control of hormone activity is caused by negative feedback mechanisms. d. Most hormones are stored in the target tissues for use later. e. Most hormones cause target tissues to change activities by changing gene activity.

a. Hormones may travel long distances to get to their target tissues. b. Continued hormone activity requires continued production and secretion. c. Control of hormone activity is caused by negative feedback mechanisms.

13. A nurse assesses a client diagnosed with adrenal hypofunction. Which client statement should the nurse correlate with this diagnosis? a. I have a terrible craving for potato chips. b. I cannot seem to drink enough water. c. I no longer have an appetite for anything. d. I get hungry even after eating a meal

a. I have a terrible craving for potato chips. The nurse correlates a clients salt craving with adrenal hypofunction.

2. A nurse cares for a client with a deficiency of aldosterone. Which assessment finding should the nurse correlate with this deficiency? a. Increased urine output b. Vasoconstriction c. Blood glucose of 98 mg/dL d. Serum sodium of 144 mEq/L

a. Increased urine output Aldosterone, the major mineralocorticoid, maintains extracellular fluid volume. It promotes sodium and water reabsorption and potassium excretion in the kidney tubules. A client with an aldosterone deficiency will have increased urine output.

7. A nurse assesses a client with Cushings disease. Which assessment findings should the nurse correlate with this disorder? (Select all that apply.) a. Moon face b. Weight loss c. Hypotension d. Petechiae e. Muscle atrophy

a. Moon face d. Petechiae e. Muscle atrophy

6. A nurse collaborates with an unlicensed assistive personnel (UAP) to provide care for a client who is prescribed a 24-hour urine specimen collection. Which statement should the nurse include when delegating this activity to the UAP? a. Note the time of the clients first void and collect urine for 24 hours. b. Add the preservative to the container at the end of the test. c. Start the collection by saving the first urine of the morning. d. It is okay if one urine sample during the 24 hours is not collected.

a. Note the time of the clients first void and collect urine for 24 hours

16. An emergency nurse cares for a client who is experiencing an acute adrenal crisis. Which action should the nurse take first? a. Obtain intravenous access. b. Administer hydrocortisone succinate (Solu-Cortef). c. Assess blood glucose. d. Administer insulin and dextrose.

a. Obtain intravenous access.

1. A nurse assesses a client with anterior pituitary hyperfunction. Which clinical manifestations should the nurse expect? (Select all that apply.) a. Protrusion of the lower jaw b. High-pitched voice c. Enlarged hands and feet d. Kyphosis e. Barrel-shaped chest f. Excessive sweating

a. Protrusion of the lower jaw c. Enlarged hands and feet d. Kyphosis e. Barrel-shaped chest f. Excessive sweating

12. A client with hyperaldosteronism is being treated with spironolactone (Aldactone) before surgery. Which precautions does the nurse teach this client? a. Read the label before using salt substitutes. b. Do not add salt to your food when you eat. c. Avoid exposure to sunlight. d. Take Tylenol instead of aspirin for pain.

a. Read the label before using salt substitutes.

Which is the priority patient teaching strategy when limited time is available? a. Setting realistic goals that have high priority for the patient b. Referring the patient to a nurse educator in private practice c. Observing more experienced nurse-teachers to learn how to teach faster and more efficiently d. Providing reading materials for the patient instead of discussing information the patient needs to learn

a. Setting realistic goals that have high priority for the patient

8. A nurse teaches a client who has been prescribed a 24-hour urine collection to measure excreted hormones. The client asks, Why do I need to collect urine for 24 hours instead of providing a random specimen? How should the nurse respond? a. This test will assess for a hormone secreted on a circadian rhythm. b. The hormone is diluted in urine; therefore, we need a large volume. c. We are assessing when the hormone is secreted in large amounts. d. To collect the correct hormone, you need to urinate multiple times.

a. This test will assess for a hormone secreted on a circadian rhythm.

. A nurse cares for a client who is prescribed vasopressin (DDAVP) for diabetes insipidus. Which assessment findings indicate a therapeutic response to this therapy? (Select all that apply.) a. Urine output is increased. b. Urine output is decreased. c. Specific gravity is increased. d. Specific gravity is decreased. e. Urine osmolality is increased. f. Urine osmolality is decreased.

a. Urine output is increased. d. Specific gravity is decreased. f. Urine osmolality is decreased

13. A nurse cares for a client with chronic hypercortisolism. Which action should the nurse take? a. Wash hands when entering the room. b. Keep the client in airborne isolation. c. Observe the client for signs of infection. d. Assess the clients daily chest x-ray.

a. Wash hands when entering the room.

hyperthyroidism post op thyroidectomy

a. assess respiratory status & admin O2 b. semi-Fowler's NOT high Fowler's c. evaluate dressing for tightness & bleeding d. evaluate client for hoarseness f. assess for hypocalcemia & tetany

The ED nurse receives a telephone call and is informed that a tornado has hit a local residential area and that numerous casualties have occurred. the victims will be brought to the ED. The nurse should take which initial action? -prepare the triage rooms -activate the emergency response plan -obtain additional supplies from the central supply department -obtain additional nursing staff to assist in treating casualties

activate the emergency response plan pg 184

The nurse enters a client's room and finds that the wastebasket is on fire. The nurse immediately assists the client out of the room. What is the next nursing action? -call for help -extinguish the fire -activate the fire alarm -confine the fire by closing the room door

activate the fire alarm pg 184

A nurse suspects elder mistreatment in which of the following patients seen in the emergency department? (Select all that apply.) a. An 85-year-old male with cardiac disease who is taking blood thinners and has multiple bruises on his arms and hands b. An 86-year-old female nursing home resident admitted to the hospital with vaginal bleeding and three large bruises on her inner thigh c. A 77-year-old woman who fell at home and broke her arm after tripping over her cat d. A 73-year-old man with a history of gastric ulcers who is vomiting blood and found to be anemic and has a low BMI e. A 69-year-old man with a history of diabetes who is admitted for diabetic foot ulcers wearing dirty clothing and smells like urine

b. An 86-year-old female nursing home resident admitted to the hospital with vaginal bleeding and three large bruises on her inner thigh e. A 69-year-old man with a history of diabetes who is admitted for diabetic foot ulcers wearing dirty clothing and smells like urine

The nurse is providing care to a client diagnosed with dementia. What option is an example of the appropriate use of implied consent by the nurse? a. Preparing to draw blood from a client's arm after asking, "Can I see your arm?" b. Changing the client's dressing when the client asks, "Will you change this bandage now?" c. Using the client's monthly allowance to buy a watch when he continuously asks for the time d. Arranging for a benign mole to be removed after the client states, "I don't like this here."

b. Changing the client's dressing when the client asks, "Will you change this bandage now?"

1. A nurse assesses clients for potential endocrine dysfunction. Which client is at greatest risk for a deficiency of gonadotropin and growth hormone? a. A 36-year-old female who has used oral contraceptives for 5 years b. A 42-year-old male who experienced head trauma 3 years ago c. A 55-year-old female with a severe allergy to shellfish and iodine d. A 64-year-old male with adult-onset diabetes mellitus

b. A 42-year-old male who experienced head trauma 3 years ago

1. A nurse cares for a client who is prescribed a drug that blocks a hormones receptor site. Which therapeutic effect should the nurse expect? a. Greater hormone metabolism b. Decreased hormone activity c. Increased hormone activity d. Unchanged hormone response

b. Decreased hormone activity

7. A nurse assesses a female client who presents with hirsutism. Which question should the nurse ask when assessing this client? a. How do you plan to pay for your treatments? b. How do you feel about yourself? c. What medications are you prescribed? d. What are you doing to prevent this from happening?

b. How do you feel about yourself? Hirsutism, or excessive hair growth on the face and body, can result from endocrine disorders. This may cause a disruption in body image, especially for female clients. The nurse should inquire into the clients body image and self-perception.

15. A nurse teaches a client with a cortisol deficiency who is prescribed prednisone (Deltasone). Which statement should the nurse include in this clients instructions? a. You will need to learn how to rotate the injection sites. b. If you work outside in the heat, you may need another drug. c. You need to follow a diet with strict sodium restrictions. d. Take one tablet in the morning and two tablets at night.

b. If you work outside in the heat, you may need another drug.

7. After teaching a client who is recovering from an endoscopic trans-nasal hypophysectomy, the nurse assesses the clients understanding. Which statement made by the client indicates a correct understanding of the teaching? a. I will wear dark glasses to prevent sun exposure. b. Ill keep food on upper shelves so I do not have to bend over. c. I must wash the incision with peroxide and redress it daily. d. I shall cough and deep breathe every 2 hours while I am awake.

b. Ill keep food on upper shelves so I do not have to bend over.

12. A nurse cares for a client who has excessive catecholamine release. Which assessment finding should the nurse correlate with this condition? a. Decreased blood pressure b. Increased pulse c. Decreased respiratory rate d. Increased urine output

b. Increased pulse

4. A nurse teaches a client with Cushings disease. Which dietary requirements should the nurse include in this clients teaching? (Select all that apply.) a. Low calcium b. Low carbohydrate c. Low protein d. Low calories e. Low sodium

b. Low carbohydrate d. Low calories e. Low sodium

9. A nurse plans care for an older adult who is admitted to the hospital for pneumonia. The client has no known drug allergies and no significant health history. Which action should the nurse include in this clients plan of care? a. Initiate Airborne Precautions. b. Offer fluids every hour or two. c. Place an indwelling urinary catheter. d. Palpate the clients thyroid gland.

b. Offer fluids every hour or two. A normal age-related endocrine change is decreased antidiuretic hormone (ADH) production. This results in a more diluted urine output, which can lead to dehydration. If no contraindications are known, the nurse should offer (or delegate) the client something to drink at least every 2 hours.

10. A nurse is caring for a client who was prescribed high-dose corticosteroid therapy for 1 month to treat a severe inflammatory condition. The clients symptoms have now resolved and the client asks, When can I stop taking these medications? How should the nurse respond? a. It is possible for the inflammation to recur if you stop the medication. b. Once you start corticosteroids, you have to be weaned off them. c. You must decrease the dose slowly so your hormones will work again. d. The drug suppresses your immune system, which must be built back up.

b. Once you start corticosteroids, you have to be weaned off them.

8. A nurse cares for a client who possibly has syndrome of inappropriate antidiuretic hormone (SIADH). The clients serum sodium level is 114 mEq/L. Which action should the nurse take first? a. Consult with the dietitian about increased dietary sodium. b. Restrict the clients fluid intake to 600 mL/day. c. Handle the client gently by using turn sheets for re-positioning. d. Instruct unlicensed assistive personnel to measure intake and output.

b. Restrict the clients fluid intake to 600 mL/day.

3. A nurse assesses a client who potentially has hyperaldosteronism. Which serum laboratory values should the nurse associate with this disorder? (Select all that apply.) a. Sodium: 150 mEq/L b. Sodium: 130 mEq/L c. Potassium: 2.5 mEq/L d. Potassium: 5.0 mEq/L e. pH: 7.28f. pH: 7.50

b. Sodium: 130 mEq/L d. Potassium: 5.0 mEq/L e. pH: 7.28f. pH: 7.50

14. A nurse teaches an older adult with a decreased production of estrogen. Which statement should the nurse include in this clients teaching to decrease injury? a. Drink at least 2 liters of fluids each day. b. Walk around the neighborhood for daily exercise. c. Bathe your perineal area twice a day. d. You should check your blood glucose before meals.

b. Walk around the neighborhood for daily exercise.

3. A nurse cares for a male client with hypopituitarism who is prescribed testosterone hormone replacement therapy. The client asks, How long will I need to take this medication? How should the nurse respond? a. When your blood levels of testosterone are normal, the therapy is no longer needed. b. When your beard thickens and your voice deepens, the dose is decreased, but treatment will continue forever. c. When your sperm count is high enough to demonstrate fertility, you will no longer need this therapy. d. With age, testosterone levels naturally decrease, so the medication can be stopped when you are 50 years old.

b. When your beard thickens and your voice deepens, the dose is decreased, but treatment will continue forever.

11. A nurse cares for a client with adrenal hyperfunction. The client screams at her husband, bursts into tears, and throws her water pitcher against the wall. She then tells the nurse, I feel like I am going crazy. How should the nurse respond? a. I will ask your doctor to order a psychiatric consult for you. b. You feel this way because of your hormone levels. c. Can I bring you information about support groups? d. I will close the door to your room and restrict visitors.

b. You feel this way because of your hormone levels.

Which patient characteristic enhances the teaching-learning process? a. moderate anxiety b. high self-efficacy c. being in the pre-contemplative stage of change d. being able to laugh about the current health problem

b. high self-efficacy

Which of the following statements made by a family caregiver would a nurse consider most indicative of elder abuse? a. "I get so frustrated because my father used to be so competent and now cannot even feed himself." b. "Mom cannot pay her own bills anymore. We went to the bank and arranged for me to have access to her checking account and help her pay the bills." c. "My dad wanders at night and I can't be bothered with him. I mix sleeping pills in his dinner so that he will fall asleep." d. "Mom asks me to do everything for her, but I think it is better if she keeps on doing as much as she is capable of."

c. "My dad wanders at night and I can't be bothered with him. I mix sleeping pills in his dinner so that he will fall asleep."

A nurse suspects that her next-door neighbor, an older woman, is a victim of elder abuse by her daughter. What is the appropriate action for the nurse to do in this situation? a. Because the neighbor is not a patient, the nurse should not get involved. b. Visit the neighbor frequently to confirm the suspicions. c. Complete a confidential report with the adult protective services in the area. d. Ask the neighbor herself if she is being abused.

c. Complete a confidential report with the adult protective services in the area.

11. A nurse evaluates laboratory results for a male client who reports fluid secretion from his breasts. Which hormone value should the nurse assess first? a. Posterior pituitary hormones b. Adrenal medulla hormones c. Anterior pituitary hormones d. Parathyroid hormone

c. Anterior pituitary hormones Breast fluid and milk production are induced by the presence of prolactin, secreted from the anterior pituitary gland.

3. A nurse cares for a client with excessive production of thyrocalcitonin (calcitonin). For which electrolyte imbalance should the nurse assess? a. Potassium b. Sodium c. Calcium d. Magnesium

c. Calcium

Which technique is MOST appropriate when using motivational interviewing with a patient who tells you that he is ready to start a weight loss program? a. Confirm that the patient is serious about losing weight b. Insists that the patient consider an organized group weight-loss program. c. Focus on the patient's strengths to support his optimism that he can successfully lose weight d. Ask a prescribed set of questions to increase the patient's awareness of his dietary behaviors

c. Focus on the patient's strengths to support his optimism that he can successfully lose weight

5. After teaching a client with acromegaly who is scheduled for a hypophysectomy, the nurse assesses the clients understanding. Which statement made by the client indicates a need for additional teaching? a. I will no longer need to limit my fluid intake after surgery. b. I am glad no visible incision will result from this surgery. c. I hope I can go back to wearing size 8 shoes instead of size 12. d. I will wear slip-on shoes after surgery to limit bending over.

c. I hope I can go back to wearing size 8 shoes instead of size 12.

15. A nurse cares for a client who is prescribed a serum catecholamine test. Which action should the nurse take when obtaining the sample? a. Discard the first sample and then begin the collection. b. Draw the blood sample after the client eats breakfast. c. Place the sample on ice and send to the laboratory immediately. d. Add preservatives before sending the sample to the lab

c. Place the sample on ice and send to the laboratory immediately.

14. A nurse cares for a client who is recovering from a hypophysectomy. Which action should the nurse take first? a. Keep the head of the bed flat and the client supine. b. Instruct the client to cough, turn, and deep breathe. c. Report clear or light yellow drainage from the nose. d. Apply petroleum jelly to lips to avoid dryness.

c. Report clear or light yellow drainage from the nose.

A nurse assesses a client who is recovering from a transsphenoidal hypophysectomy. The nurse notes nuchal rigidity. Which action should the nurse take first? a. Encourage range-of-motion exercises. b. Document the finding and monitor the client. c. Take vital signs, including temperature. d. Assess pain and administer pain medication.

c. Take vital signs, including temperature.

9. A nurse plans care for a client with Cushings disease. Which action should the nurse include in this clients plan of care to prevent injury? a. Pad the siderails of the clients bed. b. Assist the client to change positions slowly. c. Use a lift sheet to change the clients position. d. Keep suctioning equipment at the clients bedside.

c. Use a lift sheet to change the clients position.

2. A nurse plans care for a client with a growth hormone deficiency. Which action should the nurse include in this clients plan of care?a. Avoid intramuscular medications. b. Place the client in protective isolation. c. Use a lift sheet to re-position the client. d. Assist the client to dangle before rising.

c. Use a lift sheet to re-position the client.

The nurse needs to include caregivers in patient teaching primarily because (select all that apply) a. caregivers provide all the care for patients after discharge b. they might feel rejected if they are not included in the teaching c. patients have better outcomes when their caregivers are involved d. the patient may be too ill or too stressed to fully understand the teaching e. caregivers are responsible for the overall management of the patient's care

c. patients have better outcomes when their caregivers are involved d. the patient may be too ill or too stressed to fully understand the teaching

What would be the priority teaching goal for a middle-aged Hispanic woman about methods to relieve symptoms of menopause? a. precent the development of future disease b. maintain the patient's current state of health c. provide information on possible treatment options d. change the patient's beliefs about herbal supplements

c. provide information on possible treatment options

A mother calls a neighbor who is a nurse and tells the nurse that her 3-year-old daughter has just ingested liquid furniture polish. The nurse would direct the mother to take which immediate action? -induce vomiting -call an ambulance -call the Poison Control Center -bring the child to the ED

call the poison control center pg 184

tests for hypocalcemia (tetany)

chvosteks trousseus

The nurse is preparing to initiate an IV line containing a high dose of potassium chloride and plans to use a IV infusion pump. The nurse brings the pump to the bedside, prepares to plug the pump cord into the wall, and notes that no receptacle is available in the wall socket. The nurse should take which action next? -initiate the IV without the use of a pump -contact the electrical maintenance department for assistance -plug in the pump cord in the available plug above the room sink -use an extension cord form the nurses' lounge for the pump plug

contact the electrical maintenance department for assistance pg 183

Which option is an example of elder exploitation? a. A homebound client is left alone for days at a time by the caregiver. b. An older client is smacked if he doesn't eat all of his food. c. A client diagnosed with Alzheimer's disease is bathed only twice a month. d. A homebound client can only get groceries by agreeing to pay for her neighbor's groceries, too.

d. A homebound client can only get groceries by agreeing to pay for her neighbor's groceries, too.

4. A nurse cares for a client after a pituitary gland stimulation test using insulin. The clients post-stimulation laboratory results indicate elevated levels of growth hormone (GH) and adrenocorticotropic hormone (ACTH). How should the nurse interpret these results? a. Pituitary hypofunction b. Pituitary hyperfunction c. Pituitary-induced diabetes mellitus d. Normal pituitary response to insulin

d. Normal pituitary response to insulin

5. A nurse prepares to palpate a clients thyroid gland. Which action should the nurse take when performing this assessment? a. Stand in front of the client instead of behind the client. b. Ask the client to swallow after palpating the thyroid. c. Palpate the right lobe with the nurses left hand. d. Place the client in a sitting position with the chin tucked down.

d. Place the client in a sitting position with the chin tucked down.

The nurse has taught a family caregiver how to administer insulin. Evaluation of the caregiver's learning would include: a. monitoring the patient's glucose readings b. arranging for follow-up with a home care nurse c. asking the caregiver to "show back" the ability to administer insulin d. asking the caregiver what was helpful about the teaching experience

d. asking the caregiver what was helpful about the teaching experience

A patient tells the nurse that she enjoys talking with others and sharing experiences but often falls asleep when reading. Which teaching strategy would be MOST effective with this patient? a. formal lecture b. journal writing c. web-based program d. small group discussion

d. small group discussion

goiter

enlarged thyroid can be 4X the size of normal may hear a bruit with stethoscope

The nurse is giving a report to an UAP who will be caring for a client who has hand restraints. The nurse instructs the UAP to check the skin integrity of the restrained hands how frequently? -every 2 hours -every 3 hours -every 4 hours -every 30 minutes

every 30 minutes pg 184

exopthalmus

extra fat/fluid shortening of extra ocular muscles can cause blurred visions can become very dry with corneal abrasions eye drops, taping eyes at nights

Contact precautions are initiated for a client with a nosocomial infection caused by MRSA. The nurse prepares to provide colostomy care and should obtain which protective items to preform this procedure? -gloves and gown -gloves and goggles -gloves, gown, and shoe protectors -gloves, gown, goggles, and face shield

gloves, gown, goggles, and face shield pg 184

hyperthyroidism AKA

graves disease produces antibodies that go to the thyroid tissue to the receptors & causes overproduction of T3 & T4 remissions & exacerbations if not treated can lead to hypothyroidism

hyperthyroidism s/s (rabbit)

hyper hot skinny

thyrotoxicosis

increased hormone production

The community health nurse is providing a teaching session about terrorism to members of the community and is discussing information regarding anthrax. The nruse tells those attending that anthrax can be transmitted by which of the following route(s)? select all that apply. -bites from ticks or deer flies -inhalation of bacterial spores -through a cut or abrasion of the skin -direct contact with infected individual -sexual contact -ingestion of contaminated or undercooked meat

inhalation, cut/abrasion, contaminated meat pg 183/184

The nurse is reviewing a plan of care for a client with an internal radiation implant. Which intervention if noted in the plan indicates the need for revision of the plan? -wearing gloves when emptying the client's bedpan -keeping all linens in the room until the implant is removed -wearing a lead apron when proving direct care to the client -placing the client in a semiprivate room at the end of the hallway

placing the client in a semiprivate room at the end of the hallway pg 184

The nurse is caring for a client with meningitis and implements which transmission based precautions for this client? -private room or cohort client -personal respiratory protection device -private room with negative air flow -mask worn by staff when the client needs to leave the room

private room or cohort client pg 184

Adult day care centers

provide good opportunities for social interaction

Long-term care facilities

receive reimbursement from Medicaid coverage facilities. Long-term care is provided for a patient who needs 24-hour assistance for activities of daily living.

semi-Fowlers position (post thyroidectomy)

reduces stress on the suture line

The nurse obtains a prescription form a HCP to restrain a client and instructs an UAP to apply the safety device to the client. Which observation by the nurse indicates unsafe application of the safety device by the UAP? -placing a safety knot in the safety device straps -safely securing the safety device straps to the side rails -apply safety device straps that do not tighten when force is applied against them -securing so that two fingers can slid easily between the safety device and the client's skin

safely securing the safety device straps to the side rails pg 183

anterior pituitary hormones

thyroid stimulating hormone (TSH) Adrenocorticotropic hormone (ACTH) growth hormone (GH)

pretibial myxedema

tibial area extra depositis that collect in Sub Q tissue may resemble keloids

hoarseness, post-thyroidectomy

will be hoarse at first, worsening hoarseness indicates damage to the laryngeal nerve


Ensembles d'études connexes

Study Guide: Appendicular Skeleton Session Questions

View Set

L'Oreal Superday - Technical Questions

View Set

FCE Use of English Part 4 (TESTS 4-6) Paraphrasing

View Set

Chapter 28: Obstructive Pulmonary Diseases (Lewis)

View Set

Texas Life and Health - Chapter 3 Life Policy Riders, Provisions, Options and Exclusions

View Set